Analysis 1 - Mid-Sem exam - 2008-09
B.Math (Hons.)
Problem 1. Show that the ordered field R of real numbers has the Archimedean property.
Solution. We need to prove that for every x > 0, y > 0 , there exists a natural number n such that
nx > y. Let N denote the set of natural numbers. We first prove the following lemma.
Lemma: N is unbounded.
Suppose, N is bounded. By, l.u.b axiom there exists a least upper bound α such that n ≤ α for all n ∈ N.
Now, since α is the least upper bound, α − 1 is not an upper bound. Thus, there exists a k such that
k ≥ α − 1. This implies that k + 2 > α. Hence we get a contradiction. Thus, N is unbounded.
Now, given x > 0, y > 0 consider y/x. By the previous lemma, there exists a n ∈ N such that n > y/x.
Thus we have nx > y.
Problem 2. For any two subsets A, B of R , define
A+B = {a + b : a ∈ A, b ∈ B};
AB = {ab : a ∈ A, b ∈ B}
Show that if A, B are bounded above,
sup(A + B) = sup(A) + sup(B).
Give an example to show that
sup(AB) = sup(A).sup(B),
need not be true.
Solution. (a) To prove that sup(A + B) = sup(A) + sup(B).
Since, A and B are bounded, sup(A), sup(B), and sup(A + B) are finite.
claim 1: sup(A + B) ≤ sup(A) + sup(B).
Let x ∈ A + B be any arbitrary element. Then x = a + b for some a ∈ A and b ∈ B. And we know that
a ≤ sup(A) and b ≤ sup(B). Hence x = a + b ≤ sup(A) + sup(B).
Hence x ≤ sup(A) + sup(B) for all x ∈ A + B. Thus we have sup(A + B) ≤ sup(A) + sup(B).
claim 2: sup(A + B) ≥ sup(A) + sup(B).
Let > 0 be arbitrary. Then there exists an a ∈ A such that a ≥ sup(A) − /2. Similarly, there exists a
b ∈ B such that b ≥ sup(B) − /2. Then a + b ≥ sup(A) + sup(B) − .
But we have sup(A + B) ≥ a + b. Hence for all > 0 we have sup(A + B) ≥ sup(A) + sup(B) − .
Therefore we have sup(A + B) ≥ sup(A) + sup(B).
1
Thus, from claim 1 and claim 2, we have sup(A + B) = sup(A) + sup(B).
(b) To show that sup(AB) = sup(A)sup(B) is not necessarily true.
Let A = [−1, 0] and B = [−1, 0]. We have sup(A) = sup(B) = 0, and hence sup(A).sup(B) = 0.
Now, we have that 1 ∈ A.B. Thus sup(A, B) ≥ 1. Hence we cannot have sup(AB) = sup(A)sup(B).
Problem 3. Show that intervals [0, 1] and (0, 1) have same cardinality.
Solution. Let
1
H= :n∈N
n
Define the function f : (0, 1) → [0, 1] such that
0 if x = 1/2
1 if x = 1/3
f (x) =
1/(n − 2) if x = 1/n ; n ≥ 3 and x ∈ N
x if x ∈
/H
Now, define g : [0, 1] → (0, 1) as
1/2 if x=0
1/3 if x=1
f (x) =
1/(n + 2) if x = 1/n ; n ≥ 3 and x ∈ N
x if x∈/ H ∪ {0, 1}
Now note that f ◦ g = I and g ◦ f = I. Hence we have that f is a bijection, and the intervals [0, 1] and
(0, 1) have same cardinality.
Problem 4. Let {an } be a sequence of real numbers converging to a ∈ R. Define
1
bn = (a1 + a2 + · · · + an )
n
for n ≥ 1. Show that {bn } is a converging sequence converging to a.
Solution. Let > 0 be arbitrary. Since {ai } is converging sequence, we can choose a K such that
| ai − a |< ∀i ≥ K
2
Let
K
X
M= | ai − a |
i=1
2
For n ≥ K, we have
n
1 X
| bn − a | = ai − na
n i=1
n
1X
≤ | ai − a |
n i=1
K n
1X 1 X
= | ai − a | + | ai − a |
n i=1 n
i=K+1
M n−K
≤ +
n 2 n
M
≤ +
n 2
Now there exists a N such that M/n < /2 for all n ≥ N . Therefore for all n ≥ max{N.K}, we have
| bn − a |≤ + ≤ .
2 2
Since is arbitrary bn converges to a.
Problem 5. Show that every Cauchy sequence of real numbers is convergent.
Solution. Let {xn } be a cauchy sequence in R. Let > 0 be arbitrary. There exist N > 0, such that for
all m, n ≥ N , we have that |xn − xm | < /2. In particular we have |xn − xN | < /2. Or equivalently,
xn ∈ (xN − /2, xN + /2) ∀n ≥ N (1)
From this we make the following observations:
(a) For all n > N we have xn > xN − /2
(b) If xn > xN + /2, then n ∈ {1, 2, · · · N − 1}. Thus, the set of n such that xn ≥ xN + /2 is finite.
Let S = {x ∈ R : there exists infintely many n such thatxn ≥ x}. We claim that S is non-empty,
bounded above and that sup(S) is the limit of the given sequence.
Let = 1. There there exists a N such that (1) holds. Therefore xN − 1/2 belongs to S. Hence S is non
empty.
We claim that xN + 1 is an upper bound for S. If this were not true, there exists a y ∈ S such that
y > xN + 1 and xn ≥ y for infinitely many n. This implies that xn > xN + 1 for infinitely many n. This
results in a contradiction from (b). Hence xN + 1 is an upper bound for S.
By the LUB axiom, there exists a l ∈ R, such that sup(S) = l. We claim that lim xn = l. Let > 0 be
given. As l is an upper bound for S and xN − /2 ∈ S, from above, we infer that xN − /2 ≤ l. Since
l is the least upperbound for S and xN + /2 is an upper bound for S, we see that l ≤ xN + /2. Thus
we have xN − /2 ≤ l ≤ xN + /2 or
|xN − l| ≤ /2
For n ≥ N , we have
|xn − l| ≤ |xn − xN | + |xN − l|
≤ /2 + /2
Thus we have shown that lim xn = l.
3
Problem 6. Let A be a subset of R. Show that the interior of A is the largest open set contained in A.
Solution. Denote interior of A as int(A). To show that, int(A) is the largest open set contained in A.
Firstly, by definition of int(A), we have that int(A) is an open set contained in A. Let X be the largest
open set. We have that X ⊇ int(A).
Let x ∈ X. Since X is open, there exists an > 0 such that B(x, ) ⊂ X ⊂ A. From the definition of
int(A) we have X ⊆ int(A).
Hence X = int(A).
Problem 7. Suppose g : R → R is a continuous function. Define a new function g+ : R → R by
(
0 if g(x) < 0,
g+ (x) =
x otherwise.
Show that if g is continuous then g+ is continuous. Give an example where g+ is continuous but g is not
continuous.
Solution. (a) To show that g+ is continuous. Firstly, note that
g(x) | g(x) |
g+ (x) = +
2 2
claim 1 : Let x ∈ R. If g is continuous at x then | g | is continuous at x.
Let > 0 be arbitrary. Since, g is continuous at x there exists a δ > 0 such that if | x − y |< δ, then
| g(x) − g(y) |<
Now,
| g(x) | − | g(y) | ≤| g(x) − g(y) |≤
thus we have that at x, there exists a δ > 0 such that if | x − y |< δ, then
| g(x) | − | g(y) | <
Thus, | g | is continuous.
Since, linear combination of continuous functions is continuous, we have g+ is continuous.
(b) To show that if g+ is continuous, g need not be continuous.
Let g : R → R, given by (
0 if x ∈ Q
g(x) =
−1 if x ∈ Qc
Note that g is discontinuous. Now, we have g+ (x) = 0 for all x ∈ R, which is continuous. Hence if g+ is
continuous, g need not be continuous.